What is the Green's function for this specific problem?

Click For Summary
The discussion focuses on finding the Green's function for a specific differential equation with given boundary conditions. The initial approach using Legendre polynomials led to complications, particularly with Mathematica indicating the ODE was not solved correctly. The solution was refined by adjusting the Wronskian evaluation to be at y instead of s, which resolved the boundary conditions and continuity requirements for the Green's function. The final formulation satisfies the necessary conditions, including the jump discontinuity at s=y, confirming the validity of the edited solution. The problem's non-self-adjoint nature is acknowledged, affecting the symmetry of the Green's function.
member 428835

Homework Statement


Find Green's function of $$K(\phi(s)) = \phi''(s)+\cot(s)\phi'(s)+\left(2-\frac{1}{\sin(s)^2}\right)\phi(s):s\in[0,\alpha]$$
subject to boundary conditions: $$\phi|_{s=0} < \infty\\
\phi|_{s=\alpha} = 0.$$

Homework Equations


Green's function ##G## is found via variation of parameters: $$
G_1(s,y) = \frac{v_1(s)v_2(y)}{W(s)}:0<s<y<\alpha\\
G_2(s,y) = \frac{v_1(y)v_2(s)}{W(s)}:0<y<s<\alpha$$
where ##W## is the Wronskian of ##v_1,v_2## and ##v_i## is a linearly independent solution satisfying a boundary condition.

The Attempt at a Solution


Legendre polynomials of first and second kind (##P_1^1(\cos(s)),Q_1^1(\cos(s))##) solve this ODE, so we construct ##v_1## and ##v_2## from these. To satisfy the finite domain at ##s=0## we take ##v_1 = P_1^1(\cos(s))## and to satisfy ##\phi(\alpha)=0## we take the solution $$v_2(s) = \frac{\tau_2}{\tau_1}(P_1^1(\cos(s))-Q_1^1(\cos(s))):\\
\tau_1 = P_1^1(\cos(\alpha));\,\,\tau_2 = Q_1^1(\cos(\alpha)).$$
Then we find ##W(s) = -2\csc(s)##. Then we know the Green's function from above. However, when I substitute these results into the proposed method, Mathematica is suggesting the ODE is not solved. Any help is much appreciated!

Edit: Evidently the problem is solved when I take solutions of the form $$
G_1(s,y) = \frac{v_1(s)v_2(y)}{W(y)}:0<s<y<\alpha\\
G_2(s,y) = \frac{v_1(y)v_2(s)}{W(y)}:0<y<s<\alpha
$$
since now the boundaries are satisfied ##G1(s=0,y)<\infty;\, G2(s=\alpha,y)=0##. Also, ##G## is continuous: ##G2(s,y)|_{s=y}-G1(s,y)|_{s=y} = 0##, and ##G## exhibits the jump discontinuity at ##s=y##: ##G2'(s,y)|_{s=y}-G1'(s,y)|_{s=y} = 1##.

However, I should note that ##G(s,y)## satisfies the above conditions, but ##G(y,s)## does not. Can anyone comment here?
 
Last edited by a moderator:
Physics news on Phys.org
Your problem isn't self adjoint, so in general ##G(s,y)\neq G(y,s)##. By the way, given that your problem is not self adjoint, are you sure that your formulae,
joshmccraney said:
$$
G_1(s,y) = \frac{v_1(s)v_2(y)}{W(s)}:0<s<y<\alpha\\
G_2(s,y) = \frac{v_1(y)v_2(s)}{W(s)}:0<y<s<\alpha$$

are correct?

jason
 
jasonRF said:
Your problem isn't self adjoint, so in general ##G(s,y)\neq G(y,s)##. By the way, given that your problem is not self adjoint, are you sure that your formulae,are correct?

jason
Thanks for replying Jason! Good to know that fact about Green's functions. Regarding the formula, I think it's slightly incorrect. I in fact think if you evaluate the Wronskian at ##y## instead of ##s## you get the correct solution. Then I'm pretty sure the edited solution in post 1 is valid fro this problem.

Taking the proposed ##v##'s from post 1 and their Wronskian, the Green's function suggested in post 1 edit satisfies ##K(G) = 0## for ##s\neq y##, ##G## is continuous, ##G'## has a unit jump at ##s=y##, and boundary conditions are satisfied by ##G1## and ##G2##. What do you think?
 
Question: A clock's minute hand has length 4 and its hour hand has length 3. What is the distance between the tips at the moment when it is increasing most rapidly?(Putnam Exam Question) Answer: Making assumption that both the hands moves at constant angular velocities, the answer is ## \sqrt{7} .## But don't you think this assumption is somewhat doubtful and wrong?

Similar threads

  • · Replies 10 ·
Replies
10
Views
2K
  • · Replies 1 ·
Replies
1
Views
2K
  • · Replies 18 ·
Replies
18
Views
3K
  • · Replies 3 ·
Replies
3
Views
1K
Replies
9
Views
2K
Replies
3
Views
2K
Replies
1
Views
2K
  • · Replies 8 ·
Replies
8
Views
2K
  • · Replies 3 ·
Replies
3
Views
2K
Replies
7
Views
2K